1. Trang chủ
  2. » Khoa Học Tự Nhiên

Bất đẳng thức đồng bậc

9 1.5K 4

Đang tải... (xem toàn văn)

THÔNG TIN TÀI LIỆU

Bất đẳng thức đồng bậc Huỳnh Tấn Châu Trường THPT Chuyên Lương Văn Chánh, Phú Yên Trong những năm gần đây, một số đề thi học sinh giỏi Quốc gia có bài toán về bất đẳng thức. Một số các bài toán bất đẳng thức có dạng thuần nhất (đồng bậc). Nhằm giúp cho các em học sinh trong đội tuyển tiếp cận và rèn luyện kỹ năng giải quyết các bài toán này, tôi xin được trao đổi bài viết nhỏ này. 1. BẤT ĐẲNG THỨC THUẦN NHẤT (ĐỒNG BẬC) Hàm số f (x 1 , x 2 , , x n ) của các biến số thực x 1 , x 2 , , x n được gọi là hàm thuần nhất bậc k nếu với mọi số thực t ta có :f (tx 1 , tx 2 , , tx n ) = t k f (x 1 , x 2 , , x n ) Bất đẳng thức f (x 1 , x 2 , , x n ) ≥ 0, với f là hàm thuần nhất, được gọi là bất đẳng thức thuần nhất Ví dụ : Các bất đẳng thức Cauchy, bất đẳng thức Bunhiacopsky, bất đẳng thức Chebyshev là các bất đẳng thức thuần nhất. 2. MỘT SỐ BẤT ĐẲNG THỨC CHUYỂN ĐƯỢC VỀ DẠNG ĐỒNG BẬC Bài toán 1. Cho a, b, c > 0 và a + b + c = 1. Chứng minh rằng : 5 (a 2 + b 2 + c 2 ) ≤ 6 (a 3 + b 3 + c 3 ) + 1 (1) Lời giải. Với giả thiết a + b + c =1, viết lại bất đẳng thức (1) dưới dạng tương đương : 5 (a 2 + b 2 + c 2 ) (a + b + c) ≤ 6 (a 3 + b 3 + c 3 ) + (a + b + c) 3 ⇔ 5 (a 3 + b 3 + c 3 + a 2 b + a 2 c + b 2 a + b 2 c + c 2 a + c 2 b) ≤ 6 (a 3 + b 3 + c 3 ) + a 3 + b 3 + c 3 + 3 (a 2 b + a 2 c + b 2 a + b 2 c + c 2 a + c 2 b + 2abc) ⇔ 2 (a 3 + b 3 + c 3 ) + 6abc ≥ 2 (a 2 b + a 2 c + b 2 a + b 2 c + c 2 a + c 2 b) ⇔ a 3 + b 3 + c 3 + 3abc ≥ a 2 b + a 2 c + b 2 a + b 2 c + c 2 a + c 2 b ⇔ (b + c − a) (c + a − b) (a + b − c) ≤ abc(2) bất đẳng thức (2) dễ dàng chứng minh.Vậy bất đẳng thức (1) được chứng minh. Bài toán 2. (ROMANIA – BALKAN TST – 2006) Cho a, b, c > 0 và thỏa mãn a + b + c = 1. Chứng minh rằng : a 2 b + b 2 c + c 2 a ≥ 3 (a 2 + b 2 + c 2 ) (1) Lời giải. Cách 1. Biến đổi và áp dụng bất đẳng thức Cauchy – Schwarz ta có: 202 a 2 b + b 2 c + c 2 a = a 4 a 2 b + b 4 b 2 c + c 4 c 2 a ≥ (a 2 + b 2 + c 2 ) 2 a 2 b + b 2 c + c 2 a Bất đẳng thức (1) được chứng minh nếu ta chứng minh được : (a 2 + b 2 + c 2 ) 2 a 2 b + b 2 c + c 2 a ≥ 3 (a 2 + b 2 + c 2 ) ⇔ a 2 + b 2 + c 2 ≥ 3 (a 2 b + b 2 c + c 2 a) ⇔ (a 2 + b 2 + c 2 ) (a + b + c) ≥ 3 (a 2 b + b 2 c + c 2 a) (vì a + b + c = 1) ⇔ a 3 + a 2 c + b 3 + b 2 a + c 3 + c 2 b ≥ 2 (a 2 b + b 2 c + c 2 a) (2) Áp dụng bất đẳng thức AM – GM ta có :a 3 + ab 2 ≥ 2 √ a 3 .ab 2 = 2a 2 b Tương tự ta được : b 3 + bc 2 ≥ 2b 2 c, c 3 + ca 2 ≥ 2c 2 a Cộng ba bất đẳng thức trên vế theo vế ta thu được bất đẳng thức (2). Vậy bất đẳng thức (1) được chứng minh. Đẳng thức xảy ra khi và chỉ khi a = b = c. Cách 2. Với a + b + c = 1, ta có : a 2 b + b 2 c + c 2 a ≥ 3 (a 2 + b 2 + c 2 ) (1) ⇔  a 2 b + b 2 c + c 2 a  (a + b + c) ≥ 3 (a 2 + b 2 + c 2 ) ⇔ a 2 + b 2 + c 2 + a 3 b + b 3 c + c 3 a + a 2 c b + b 2 a c + c 2 b a ≥ 3 (a 2 + b 2 + c 2 ) ⇔ a 3 b + b 3 c + c 3 a + a 2 c b + b 2 a c + c 2 b a ≥ 2 (a 2 + b 2 + c 2 ) (2) Áp dụng bất đẳng thức AM – GM : a 2 c b + bc ≥ 2ac; b 2 a c + ac ≥ 2ba; c 2 b a + ba ≥ 2bc ⇒ a 2 c b + b 2 a c + c 2 b a ≥ ab+bc+ca ⇒ a 3 b + b 3 c + c 3 a + a 2 c b + b 2 a c + c 2 b a ≥ a 3 b + b 3 c + c 3 a +ab+bc+ca (3) Áp dụng bất đẳng thức AM – GM : a 3 b + ab ≥ 2a 2 , b 3 c + bc ≥ 2b 2 , c 3 a + ca ≥ 2c 2 ⇒ a 3 b + b 3 c + c 3 a + ab + bc + ca ≥ 2 (a 2 + b 2 + c 2 ) (4) Từ (3) và (4) suy ra : ⇔ a 3 b + b 3 c + c 3 a + a 2 c b + b 2 a c + c 2 b a ≥ 2 (a 2 + b 2 + c 2 ) (đpcm) Bài toán 3. (BALAN – 2010) Cho a, b, c là các số thực dương thỏa mãn a 4 + b 4 + c 4 ≥ a 3 + b 3 + c 3 Chứng minh rằng : a 3 √ b 4 + b 2 c 2 + c 4 + b 3 √ c 4 + c 2 a 2 + a 4 + c 3 √ a 4 + a 2 b 2 + b 4 ≥ √ 3 Lời giải. Từ giả thiết suy ra a 4 + b 4 + c 4 a 3 + b 3 + c 3 ≥ 1 nên ta qui bài toán về việc chứng minh bất đẳng thức đồng bậc là : a 3 √ b 4 + b 2 c 2 + c 4 + b 3 √ c 4 + c 2 a 2 + a 4 + c 3 √ a 4 + a 2 b 2 + b 4 ≥ √ 3. a 4 + b 4 + c 4 a 3 + b 3 + c 3 Sử dụng kĩ thuật ghép đối xứng, ta sẽ chỉ ra rằng : a 3 √ b 4 + b 2 c 2 + c 4 ≥ √ 3a 4 a 3 + b 3 + c 3 ⇔ 3a 2 (b 4 + b 2 c 2 + c 4 ) ≤ (a 3 + b 3 + c 3 ) 2 (1) Áp dụng bất đẳng thức AM – GM ta có : 3a 2 b 4 = 3.ab.ab.b 2 ≤ a 3 b 3 + a 3 b 3 + b 6 3a 2 c 4 = 3.ac.ac.c 2 ≤ a 3 c 3 + a 3 c 3 + c 6 ; 3a 2 b 2 c 2 = 3.a 2 .bc.bc ≤ a 6 + b 3 c 3 + b 3 c 3 Cộng các bất đẳng thức trên vế theo vế ta thu được bất đẳng thức (1) 203 Do đó a 3 √ b 4 + b 2 c 2 + c 4 ≥ √ 3a 4 a 3 + b 3 + c 3 Dấu bằng xảy ra khi và chỉ khi a = b = c = 1. 3. MỘT SỐ KĨ THUẬT CHỨNG MINH BẤT ĐẲNG THỨC ĐỒNG BẬC Bài toán 4. (ALBANIA – 2002) Cho a, b, c > 0. Chứng minh rằng : 1 + √ 3 3 √ 3 (a 2 + b 2 + c 2 )  1 a + 1 b + 1 c  ≥ a + b + c + √ a 2 + b 2 + c 2 Lời giải. Cách 1. 1 + √ 3 3 √ 3 (a 2 + b 2 + c 2 )  1 a + 1 b + 1 c  ≥ a + b + c + √ a 2 + b 2 + c 2 (1) Khi thay (a, b, c ) bởi (ta, tb, tc) thì bất đẳng thức không thay đổi, do đó không mất tổng quát giả sử a 2 + b 2 + c 2 = 1 Khi đó (1) : 1 + √ 3 3 √ 3  1 a + 1 b + 1 c  ≥ a + b + c + 1 ⇔ 1 + √ 3 3 √ 3  1 a + 1 b + 1 c  (a + b + c) ≥ (a + b + c) 2 + (a + b + c)(2) Theo bất đẳng thức AM – GM: 1 + √ 3 3 √ 3  1 a + 1 b + 1 c  (a + b + c) ≥ 1 + √ 3 3 √ 3 .9 = √ 3 + 3 (3) Đặt X = a + b + c ≤  3 (a 2 + b 2 + c 2 ) = √ 3 (bất đẳng thức Bunhiacốpxki). Suy ra 0 < X ≤ √ 3 Do đó (a + b + c) 2 + (a + b + c) = X 2 + X ≤ 3 + √ 3 (4) Từ (3) và (4) suy ra bất đẳng thức (2). Vậy bất đẳng thức (1) được chứng minh. Cách 2. Ta có : a + b + c ≤  3 (a 2 + b 2 + c 2 ) ⇒ a + b + c + √ a 2 + b 2 + c 2 ≤  1 + √ 3   (a 2 + b 2 + c 2 ) Bất đẳng thức (1) được chứng minh nếu ta chứng minh được: 1 + √ 3 3 √ 3 (a 2 + b 2 + c 2 )  1 a + 1 b + 1 c  ≥  1 + √ 3   (a 2 + b 2 + c 2 ) ⇔ 1 3 √ 3 √ a 2 + b 2 + c 2  1 a + 1 b + 1 c  ≥ 1 (2) Theo bất đẳng thức AM – GM : 1 3 √ 3 √ a 2 + b 2 + c 2  1 a + 1 b + 1 c  ≥ 1 3 √ 3  3 3 √ a 2 b 2 c 2 .3 3  1 abc = 1 Bài toán 5. (IRAN – 2010) Cho a, b, c > 0. Chứng minh rằng : 1 a 2 + 1 b 2 + 1 c 2 + 1 (a + b + c) 2 ≥ 7 25  1 a + 1 b + 1 c + 1 a + b + c  2 (1) Lời giải. Nhận xét : Bất đẳng thức trên là bất đẳng thức thuần nhất. Khi ta thay (a; b; c) bởi (ta; tb; tc) thì bất đẳng thức không thay đổi. Do đó không mất tính tổng quát, giả sử a + b + c = 1 (a, b, c > 0). Bất đẳng thức cần chứng minh viết lại : 1 a 2 + 1 b 2 + 1 c 2 + 1 ≥ 7 25  1 a + 1 b + 1 c + 1  2 (2) 204 Đặt 1 a = x; 1 b = y; 1 c = z (x; y; z > 0) ⇒ x + y + z = 1 a + 1 b + 1 c ≥ 9 a + b + c = 9 Bất đẳng thức (2) : x 2 + y 2 + z 2 + 1 ≥ 7 25 (x + y + z + 1) 2 Do x 2 + y 2 + z 2 ≥ 1 3 (x + y + z) 2 nên bất đẳng thức trên được chứng minh nếu ta chứng minh được : 1 3 (x + y + z) 2 + 1 ≥ 7 25 (x + y + z + 1) 2 Đặt t = x + y + z ⇒ t ≥ 9. Ta cần chứng minh : 1 3 t 2 + 1 ≥ 7 25 (t + 1) 2 ⇔ 4t 2 − 42t + 54 ≥ 0 ⇔ (t − 9)  t − 3 2  ≥ 0 Điều này hoàn toàn đúng ∀t ≥ 9. Do đó bài toán đã giải xong. Đẳng thức xảy ra khi và chỉ khi a = b = c. Bài toán 6. Chứng minh rằng trong tam giác nhọn ABC có độ dài ba cạnh là a, b, c ta luôn có : (a + b + c) 2  1 a 2 + b 2 − c 2 + 1 c 2 + b 2 − a 2 + 1 a 2 + c 2 − b 2  ≥ 27 (1) Lời giải. Khi thay (a; b; c) bởi (ta; tb; tc) thì bất đẳng thức (1) không thay đổi, nên không mất tổng quát giả sử a + b + c = 3. Khi đó (1) :  1 a 2 + b 2 − c 2 + 1 c 2 + b 2 − a 2 + 1 a 2 + c 2 − b 2  ≥ 3 (2) Theo định lí hàm số cosin ta có : c 2 = a 2 +b 2 −2abcosC ⇒ a 2 +b 2 −c 2 = 2abcosC Bất đẳng thức (2) trở thành : 1 2abcosC + 1 2bccosA + 1 2accosB ≥ 3 ⇔ c 2abccosC + a 2abccosA + b 2abccosB ≥ 3⇔ R abc (tan A + tan B + tan C) ≥ 3 Áp dụng bất đẳng thức AM – GM : 3 = a + b + c ≥ 3 3 √ abc ⇒ abc ≤ 1⇒ R abc ≥ R Mặt khác 3 = a + b + c = 2R (sin A + sin B + sin C) ≤ 2R. 3 √ 3 2 = 3 √ 3.R ⇒ R ≥ 1 √ 3 ⇒ R abc ≥ 1 √ 3 (3) Tam giác ABC có 3 góc nhọn ta luôn có tan A + tan B + tan C ≥ 3 √ 3 (4) Từ (3) và (4) suy ra R abc (tan A + tan B + tan C) ≥ 3(đpcm) Đẳng thức xảy ra khi và chỉ khi tam giác ABC là tam giác đều. Bài toán 7. Cho a, b, c là các số thực dương. Chứng minh rằng : 3  (a + b) 2 (b + c) 2 (c + a) 2 abc ≥ 4 (a + b + c) 3 (1) Lời giải. Cách 1. Khi thay (a; b; c) bởi (ta; tb; tc) thì bất đẳng thức (1) không thay đổi, nên không mất tổng quát giả sử a + b + c = 3. (1) : 3  (a + b) 2 (b + c) 2 (c + a) 2 abc ≥ 4 ⇔ (a + b) 2 (b + c) 2 (c + a) 2 ≥ 64abc ⇔ (a + b) (b + c) (c + a) ≥ 8 √ abc (2) Vì a + b + c = 3 nên (a + b) (b + c) (c + a) = (3 − c) (3 − a) (3 − b) = = 27 − 9 (a + b + c) + 205 3 (ab + bc + ca) − abc = 27 − 9.3 + 3 (ab + bc + ca) − abc = 3 (ab + bc + ca) − abc Áp dụng bất đẳng thức quen thuộc : (x + y + z) 2 ≥ 3 (xy + yz + zx) , ∀x, y, z ∈ R Ta có : (ab + bc + ca) 2 ≥ 3abc (a + b + c) = 9abc ⇒ ab+bc+ca ≥ 3 √ abc Do đó (a + b) (b + c) (c + a) ≥ 9 √ abc − abc = 8 √ abc + √ abc  1 − √ abc  Theo bất đẳng thức AM – GM ta có : 3 = a + b + c ≥ 3 3 √ abc ⇒ 1 ≥ abc ⇒ 1 − √ abc ≥ 0 Suy ra : (a + b) (b + c) (c + a) ≥ 8 √ abc. Bất đẳng thức (2) được chứng minh, đẳng thức xảy ra khi và chỉ khi a = b = c = 1. Vậy bất đẳng thức (1) được chứng minh, đẳng thức xảy ra khi và chỉ khi a = b = c . Cách 2. Chuẩn hóa a + b + c = 3 4 . Bất đẳng thức (1) trở thành : 3  (a + b) 2 (b + c) 2 (c + a) 2 abc ≥ 1 ⇔ (a + b) 2 (b + c) 2 (c + a) 2 ≥ abc. (2) Biến đổi và áp dụng bất đẳng thức AM – GM ta được : (a + b) 2 (b + c) 2 (c + a) 2 = [(a + b + c) (ab + bc + ca) − abc] 2 =  8 9 (a + b + c) (ab + bc + ca) + 1 9 (a + b + c) (ab + bc + ca) − abc  2 ≥  8 9 . 3 4 (ab + bc + ca) + 1 9 .3 3 √ abc.3 3  (abc) 2 − abc  2 =  2 3 (ab + bc + ca)  2 = 4 9 (ab + bc + ca) 2 ≥ 4 9 .3abc (a + b + c) = 4 9 .3abc. 3 4 = abc. (đpcm) Bài toán 8. (NHẬT BẢN – 1997) Cho a, b, c là các số thực dương. Chứng minh bất đẳng thức : (b + c − a) 2 (b + c) 2 + a 2 + (c + a − b) 2 (c + a) 2 + b 2 + (a + b − c) 2 (a + b) 2 + c 2 ≥ 3 5 Lời giải. Nhận xét : Bài toán này có trong cuốn sách “ Tuyển tập các bài toán từ những cuộc thị tại Trung Quốc ” , được giải khá phức tạp bằng cách sử dụng bất đẳng thức Schur. Ở đây tôi đưa hai lời giải khá đẹp, đặc biệt là lời giải thứ hai khá độc đáo nhờ việc sử dụng tính đồng bậc của các biểu thức tham gia trong bất đẳng thức. Cách 1. Đặt      2x = b + c − a 2y = c + a −b 2z = a + b − c ⇔      a = y + z b = z + x c = x + y bất đẳng thức ⇔ 4x 2 (2x + y + z) 2 + (y + z) 2 + 4y 2 (2y + z + x) 2 + (z + x) 2 + 4z 2 (2z + x + y) 2 + (x + y) 2 ≥ 3 5 ⇔ x 2 2x 2 + y 2 + z 2 + 2xy + 2xz + 2yz + y 2 2y 2 + x 2 + z 2 + 2xy + 2yz + 2zx + z 2 2z 2 + x 2 + y 2 + 2xy + 2yz + 2zx ≥ 3 10 Do 2xy ≤ x 2 + y 2 , 2yz ≤ y 2 + z 2 , 2zx ≤ x 2 + z 2 Nên VT ≥ x 2 4x 2 + 3y 2 + 3z 2 + y 2 4y 2 + 3z 2 + 3x 2 + 206 z 2 4z 2 + 3x 2 + 3y 2 Đặt x1= x2 , y1 = y2 , z1 = z2 , x1 , y1 , z1 > 0 VT ≥ x 1 4x 1 + 3y 1 + 3z 1 + y 1 4y 1 + 3z 1 + 3x 1 + z 1 4z 1 + 3x 1 + 3y 1 Các phân thức ở vế phải có tử số và mẫu số đồng bậc, không mất tổng quát, giả sử x1 + y1 +z1 = 1. VT ≥ x 1 x 1 + 3 + y 1 y 1 + 3 + z 1 z 1 + 3 Đặt f(t) = t t + 3 , t > 0,f / (t) = 3 (t + 3) 2 , f // (t) = −6 (t + 3) 3 < 0 ⇒ f(t) là hàm lồi trên (0, +∞). Áp dụng bất đẳng thức hàm lồi ta có : f (x 1 )+f (y 1 )+f (z 1 ) ≥ 3.f  x 1 + y 1 + z 1 3  = 3f  1 3  ⇒ V T ≥ 3. 1 3 1 3 + 3 = 3 10 (đpcm) Cách2. bất đẳng thức ⇔ (b + c − a) 2 (b + c) 2 + a 2 −1 + (c + a − b) 2 (c + a) 2 + b 2 −1 + (a + b − c) 2 (a + b) 2 + c 2 −1 ≥ − 12 5 ⇔ (b + c) a (b + c) 2 + a 2 + (c + a) b (c + a) 2 + b 2 + (a + b) c (a + b) 2 + c 2 ≤ 6 5 Các phân thức ở vế trái có tử số và mẫu số đồng bậc, không mất tổng quát, giả sử a + b + c = 1. Bất đẳng thức viết lại : (1 − a) a 1 − 2a + 2a 2 + (1 − b) b 1 − 2b + 2b 2 + (1 − c) c 1 − 2c + 2c 2 ≤ 6 5 Áp dụng bất đẳng thức AM – GM: 2a (1 −a) ≤ (a + 1) 2 4 Suy ra : 1 − 2a + 2a 2 ≥ 1 − (a + 1) 2 4 = (1 − a) (3 + a) 4 ⇒ (1 − a) a 1 − 2a + 2a 2 ≤ (1 − a) a (1 − a) (3 + a) 4 = 4a 3 + a Tương tự : (1 − b) b 1 − 2b + 2b 2 ≤ 4b 3 + b , (1 − c) c 1 − 2c + 2c 2 ≤ 4c 3 + c Để chứng minh bất đẳng thức đề bài ta cần chứng minh : 4a 3 + a + 4b 3 + b + 4c 3 + c ≤ 6 5 ⇔ 1 3 + a + 1 3 + b + 1 3 + c ≥ 9 10 Áp dụng bất đẳng thức AM – GM: (3 + a + 3 + b + 3 + c)  1 3 + a + 1 3 + b + 1 3 + c  ≥ 9 Suy ra 10  1 3 + a + 1 3 + b + 1 3 + c  ≥ 9 Do đó 1 3 + a + 1 3 + b + 1 3 + c ≥ 9 10 (đpcm) Bài toán 9. (MOLDOVA – 1999) Cho a, b, c > 0. Chứng minh rằng : ab c(c + a) + bc a(a + b) + ca b(b + c) ≥ a a + c + b b + a + c c + b Lời giải. Các vế của bất đẳng thức là các biểu thức cùng bậc (bậc không). Không mất tổng quát, ta có thể giả sử rằng abc = 1. Khi đó bất đẳng thức cần chứng minh trở thành (b + c)(b + a) c 2 + (c + a)(c + b) a 2 + (a + b)(a + c) b 2 ≥ ≥ (b + a)(b + c) bc + (c + a)(c + b) ca + (a + b)(a + c) ab ⇔ (ab + bc + ca)  1 c 2 + 1 a 2 + 1 b 2  +  b 2 c 2 + c 2 a 2 + a 2 b 2  ≥ ≥ (ab + bc + 207 ca)  1 bc + 1 ca + 1 ab  +  b c + c a + a b  Do 1 a 2 + 1 b 2 + 1 c 2 ≥ 1 bc + 1 ca + 1 ab và b 2 c 2 + c 2 a 2 + a 2 b 2 ≥ b c + c a + a b nên ta có đpcm. Dấu bằng xảy ra khi và chỉ khi a=b=c. Bài toán 10. (VMO – 2004 BẢNG A) Xét các số thực dương x, y, z thỏa mãn điều kiện : (x + y + z) 3 = 32xyz Hãy tìm giá trị nhỏ nhất và giá trị lớn nhất của biểu thức :P = x 4 + y 4 + z 4 (x + y + z) 4 Lời giải. Cách 1. Nhận xét rằng vớiα là một số thực dương tùy ý, ta luôn có : P(x, y, z) = P (αx, αy, αz)và nếu x, y, z thỏa mãn điều kiện của đề bài thì αx, αy, αz cũng thỏa mãn các điều kiện đó. Vì thế không mất tổng quát, có thể giả sử x+y+z=4, khi đó xyz=2 bài toán trở thành : Tìm giá trị nhỏ nhất và giá trị lớn nhất của biểu thức P = 1 256 (x 4 + y 4 + z 4 ) khi x, y, z>0 thay đổi sao cho x+y+z=4 , và xyz=2 Đặt Q=x4+y4+z4 và t=xy+yz+zx Ta có Q=(x2+y2+z2)2–2(x2y2+y2z2+z2x2) Q = (4 2 − 2t) 2 − 2 [t 2 − 2xyz(x + y + z)] Q=2t2–64t+44+32=2(t2–32t+144) (1) Từ giả thiết ta có: y + z = 4 −x, yz = 2 x (2) Do đó t = x(4 − x) + 2 x (3) Áp dụng bất đẳng thức AM – GM: y + z ≥ 2 √ yz ⇒ (4 −x) 2 ≥ 8 x ⇔ x 3 −8x 2 + 16x −8 ≥ 0 ⇔ (x −2) (x 2 − 6x + 4) ≥ 0⇔ 3 − √ 5 ≤ x ≤ 2 , do x ∈ (0; 4) Xét hàm số t = x(4 −x) + 2 x trên đoạn  3 − √ 5; 2  ta có : t  (x) = −2(x − 1) (x 2 − x − 1) x 2 ; t  (x) = 0 ⇔ x = 1 ∨ x =  1 ± √ 5  2 Suy ra 5 ≤ t ≤ 5 √ 5 − 1 2 Vì hàm số f (t) = t 2 − 32t + 144 nghịch biến trên khoảng (0; 16) và vì  5; 5 √ 5 − 1 2  ⊂ (0; 16) nên trên  5; 5 √ 5 − 1 2  ta có : min f (t) = f  5 √ 5 − 1 2  = 383 − 165 √ 5 2 , max f (t) = f (5) = 9 Kết hợp với (1) ta được : min Q = 383 − 165 √ 5 , max Q = 18 Vậy min P = 383 − 165 √ 5 256 , đạt được chẳng hạn khi x = 3 − √ 5, y = z = 1 + √ 5 2 max P = 9 128 , đạt được chẳng hạn khi x = 2, y = z = 1 Cách 2. Không mất tổng quát, ta giả sử x + y + z = 1 và từ giả thiết suy ra xyz = 1 32 . Đặt t = ab + bc + ca = xy + yz + zx x 4 +y 4 +z 4 = (1 − 2t) 2 −2 (t 2 − 2xyz) = 2(1 −t) 2 − 7 8 Thế nên để tìm giá trị lớn nhất, nhỏ nhất của P, ta cần tìm giá trị lớn nhất, nhỏ nhất của t. t = xy + yz + zx = y(1 −y) + 1 32y do x + z = 1 – y và xz = 1 32y và do (x + z) 2 ≥ 4xz 208 nên (1 −y) 2 ≥ 1 8y , giải bất phương trình bậc ba này cho ta nghiệm 1 2 ≥ y ≥ 3 − √ 5 4 , ta chỉ cần chứng minh t(y) nghịch biến trên khoảng  3 − √ 5 4 ; 1 2  và do đó ta có : t  3 − √ 5 4  ≥ t(y) ≥ t  1 2  . Từ đó tìm được giá trị lớn nhất, nhỏ nhất của P. 4. BÀI TẬP RÈN LUYỆN Bài toán 11. (USA – 2003) Cho a, b, c > 0. Chứng minh rằng : (2a + b + c) 2 2a 2 + (b + c) 2 + (2b + c + a) 2 2b 2 + (c + a) 2 + (2c + a + b) 2 2c 2 + (a + b) 2 ≤ 8 Bài toán 12. Chứng minh rằng nếu x, y, z > 0 thì ta có : x 4 + y 4 + z 4 + (x + y + z) 4 27 ≥ 2 (x 2 y 2 + y 2 z 2 + z 2 x 2 ) Bài toán 13. Cho a, b, c là các số thực dương. Chứng minh bất đẳng thức : (a + b − c) 3 2c 3 + (a + b) 3 + (a + c − b) 3 2b 3 + (c + a) 3 + (b + c − a) 3 2a 3 + (b + c) 3 ≥ 3 10 Bài toán 14. Cho a, b, c là các số thực dương. Chứng minh bất đẳng thức : 7 (a + b + c) (ab + bc + ca) ≤ 9abc + 2(a + b + c) 3 Bài toán 15. Hãy xác định số thực dương λ lớn nhất sao cho bất đẳng thức : a 4 + b 4 + c 4 + abc (a + b + c) ≥ λ(ab + bc + ca) 2 đúng với mọi số thực dương a, b, c. Bài toán 16. Hãy xác định số thực dươngλ lớn nhất sao cho bất đẳng thức : a 3 + b 3 + c 3 λabc + (a + b + c) 2 a 2 + b 2 + c 2 ≥ 3 λ + 1 3 đúng với mọi số thực dương a, b, c. Bài toán 17. Cho các số thực dương a, b, c. Chứng minh rằng: √ a + b + c + √ a b + c + √ a + b + c + √ b c + a + √ a + b + c + √ c a + b ≥ 9 + 3 √ 3 2 √ a + b + c Bài toán 18. (ROMANIA TST – 2001) Giả sử a, b, c là độ dài các cạnh của một tam giác. Chứng minh bất đẳng thức: (b + c − a) (a + c − b) + (a + c − b) (a + b − c) + (a + b − c) (b + c − a) ≤ √ abc  √ a + √ b + √ c  Bài toán 19. (VMO – 2002 BẢNG B) Cho a, b, c là ba số thực tùy ý. Chứng minh rằng : 6 (a + b + c) (a 2 + b 2 + c 2 ) ≤ 27abc+ 10(a 2 + b 2 + c 2 ) 3 2 Hỏi dấu bằng xảy ra khi nào ? 209 Bài toán 20. (VMO – 2002 BẢNG A) Giả sử a, b, c là các số thực sao cho đa thức P (x) = x 3 + ax 2 + bx + c có ba nghiệm thực (các nghiệm không nhất thiết đôi một khác nhau) Chứng minh rằng : 12ab + 27c ≤ 6a 3 + 10(a 2 − 2b) 3 2 Hỏi dấu đẳng thức xảy ra khi nào ? TÀI LIỆU THAM KHẢO [1]. Nguyễn Văn Mậu, Bất đẳng thức – Định lí và áp dụng, Nhà xuất bản Giáo dục 2006 [2]. Phan Đức Chính, Bất đẳng thức, Nhà xuất bản Giáo dục 1993 [3]. Phạm Văn Thuân, Các chuyên đề Toán học trong hệ THPT chuyên 2005 [3]. G.H. Hardy, J.E.Littlewood, G.Polya, Bất đẳng thức, Nhà xuất bản Đại Học Quốc Gia Hà Nội 2002 [4]. Các bài Thi Olympic Toán THPT Việt Nam (1990 – 2006), Nhà xuất bản Giáo dục 2007 [5]. Các nguồn tài liệu trên Internet. [6]. Tạp chí Toán học và Tuổi trẻ 210 . , x n ) Bất đẳng thức f (x 1 , x 2 , , x n ) ≥ 0, với f là hàm thuần nhất, được gọi là bất đẳng thức thuần nhất Ví dụ : Các bất đẳng thức Cauchy, bất đẳng thức Bunhiacopsky, bất đẳng thức Chebyshev. dụng bất đẳng thức AM – GM ta có :a 3 + ab 2 ≥ 2 √ a 3 .ab 2 = 2a 2 b Tương tự ta được : b 3 + bc 2 ≥ 2b 2 c, c 3 + ca 2 ≥ 2c 2 a Cộng ba bất đẳng thức trên vế theo vế ta thu được bất đẳng thức. ra bất đẳng thức (2). Vậy bất đẳng thức (1) được chứng minh. Cách 2. Ta có : a + b + c ≤  3 (a 2 + b 2 + c 2 ) ⇒ a + b + c + √ a 2 + b 2 + c 2 ≤  1 + √ 3   (a 2 + b 2 + c 2 ) Bất đẳng thức

Ngày đăng: 14/07/2015, 12:06

Xem thêm: Bất đẳng thức đồng bậc

TỪ KHÓA LIÊN QUAN

w